X



トップページ数学
625コメント682KB
現代数学の系譜 工学物理雑談 古典ガロア理論も読む48
■ このスレッドは過去ログ倉庫に格納されています
0001現代数学の系譜 雑談 古典ガロア理論も読む ◆e.a0E5TtKE
垢版 |
2017/12/14(木) 06:50:59.72ID:oVKNFyGV
“現代数学の系譜 物理工学雑談 古典ガロア理論も読む”

数学セミナー時枝記事は、過去スレ39 で終わりました。
39は、別名「数学セミナー時枝記事の墓」と名付けます。

皆さまのご尽力で、伝統あるガロアすれは、
過去、数学板での勢いランキングで、常に上位です。(勢い1位の時も多い(^^ )

このスレは、現代数学のもとになった物理工学の雑談スレとします。たまに、“古典ガロア理論も読む”とします。
それで良ければ、どうぞ。
後でも触れますが、基本は私スレ主のコピペ・・、まあ、言い換えれば、スクラップ帳ですな〜(^^

話題は、散らしながらです。時枝記事は、気が向いたら、たまに触れますが、それは私スレ主の気ままです。
“時枝記事成立”を支持する立場からのカキコや質問は、基本はスルーします。それはコピペで流します。気が向いたら、忘れたころに取り上げます。

なお、
小学レベルとバカプロ固定
サイコパスのピエロ(不遇な「一石」https://textream.yahoo.co.jp/personal/history/comment?user=_SrJKWB8rTGHnA91umexH77XaNbpRq00WqwI62dl 表示名:ムダグチ博士 Yahoo! ID/ニックネーム:hyperboloid_of_two_sheets (Yahoo!でのあだ名が、「一石」)
(参考)http://blog.goo.ne.jp/grzt9u2b/e/c1f41fcec7cbc02fea03e12cf3f6a00e サイコパスの特徴、嘘を平気でつき、人をだまし、邪悪な支配ゲームに引きずり込む 2007年04月06日
High level people
低脳幼稚園児のAAお絵かき
お断り!
小学生がいますので、18金よろしくね!(^^

High level people は自分達で勝手に立てたスレ28へどうぞ!sage進行推奨(^^;
また、スレ43は、私が立てたスレではないので、私は行きません。そこでは、私はスレ主では無くなりますからね。このスレに不満な人は、そちらへ。 http://rio2016.2ch.net/test/read.cgi/math/1506152332/
旧スレが512KBオーバー(間近)で、新スレ立てる
(スレ主の趣味で上記以外にも脱線しています。ネタにスレ主も理解できていないページのURLも貼ります。関連のアーカイブの役も期待して。)
0501132人目の素数さん
垢版 |
2017/12/24(日) 15:20:25.56ID:7MvmOIII
おっちゃんです。
>>490
>・いま、補集合 R−Bfを場合分けすると
> 1)有限個であれば、Bfが、「ある開区間(a, b) 上でリプシッツ連続である」は自明
>  2)可算無限個であっても、それが、ある区間(c,d)などに偏在していれば、「ある開区間(a, b) 上でリプシッツ連続である」も自明
>  3)この定理で、クリティカルなのは、可算無限個が、(例えば有理数などのように)R中に稠密分散されているとき。
>    言い換えれば、孤立する1点から成る集合で、R中に稠密分散されている例として、有理数や代数的数があるが、
>    もし、このような状態があれば、「ある開区間(a, b) 」は取れないから、それは反例となる。
>  4)つまり、この定理が成立すれば、定理の前提であるディニ微分関連の部分(それはリプシッツ連続とも関係している)で、「”< +∞”を満たさない」部分は
>  「R中に稠密分散され得ない」ということになる(∵R中に稠密分散される状態が実現すれば、「(a, b) 上でリプシッツ連続である」が言えない)

>>489
>(>>303より)
>”定理1.7 (422 に書いた定理)
>f : R → R とする.
の部分から、fは実数直線Rを定義域、かつRの部分集合を値域とする実関数であることが分かる。
実関数 f:R→R が或る開区間 (a,b) でリプシッツ連続になるのは
或る正の実数Kが存在して、任意の (a,b) の2点 x,y について |f(x)−f(y)|≦K|x−y| となるとき
だから、リプシッツ連続の反例になっていない。
0503132人目の素数さん
垢版 |
2017/12/24(日) 17:27:53.36ID:ThBjkOXn
>>490
>補集合 R−Bf が内点を持たない閉集合の高々可算和で被覆できる とは何だろうか?

何度も同じことを書くが、集合 A が内点を持たない閉集合の高々可算和で被覆できるとき、
A のことを「第一類集合」と呼ぶのである。つまり、

質問「補集合 R−Bf が内点を持たない閉集合の高々可算和で被覆できる とは何だろうか? 」
回答「 R―Bf は第一類集合である、ということだ」

ということである。ま、これでは単なる言葉の置き換えに過ぎないのだが、
権威主義のスレ主には、この書き方の方がヘンなイチャモンをつけにくいだろうw


>>490
>・いま、考えている通常のRにおいて、「内点を持たない閉集合」とは、孤立する1点から成る集合にほかならない

今回のスレ主の話の中では本質的ではないが、この発言そのものは間違っているので指摘しておく。

孤立点だけで構成された集合(すなわち離散集合)は高々可算無限集合にしかならないが、
内点を持たない閉集合であって非可算無限集合であるものが存在する。
たとえば、カントール集合は内点を持たない閉集合だが、カントール集合は非可算無限集合である。

また、一点集合 {p} は常に「内点を持たない閉集合」であるが、カントール集合は非可算無限集合なので、
カントール集合を「一点集合の可算無限和で被覆する」という芸当も不可能でる。

よって、「ほかならない」というスレ主の言い方は間違っている。
0504132人目の素数さん
垢版 |
2017/12/24(日) 17:38:05.69ID:ThBjkOXn
>>497
>だから、(A)”a dense uncountable subset”で、リプシッツ連続は満たさないは、実現できている
>では、なぜ、(B)”内点を持たない閉集合の高々可算和”は、実現することができないのか?
>これを見極めない限り、素人の証明を読んでも仕方が無いと思う

・ なぜ (B) では実現不可能かというと、例の定理に抵触するからだよw

・ なぜ例の定理が成り立つかというと、ベールのカテゴリ定理を使ってるからだよ。

・ ベールのカテゴリ定理に帰着させるために、技術的には1つの補題が必要になり、それが「補題1.5」だよ。

・ 例の定理の証明とは無関係に、(B) で実現不可能な理由をスレ主が独自に探っていっても、
  結局はベールのカテゴリ定理に帰着させるハメになり、例の定理の証明と同じことをするハメになるだろうw

この4項目の見極めで十分だろ。
そろそろ例の証明を読んでみたらどうだね。
0505132人目の素数さん
垢版 |
2017/12/24(日) 20:49:26.73ID:ndfap2+C
>>490
>・いま、考えている通常のRにおいて、「内点を持たない閉集合」とは、孤立する1点から成る集合にほかならない
違うよ
境界点だけということ
境界点が孤立してなくてはいけないわけではない
0506現代数学の系譜 雑談 古典ガロア理論も読む ◆e.a0E5TtKE
垢版 |
2017/12/24(日) 20:59:06.39ID:Q5UHveEY
>>497 関連

無理数で微分可能で、有理数のみ微分不可能という
函数の構成があったので、貼っておく(^^

http://www.mathcounterexamples.net/a-continuous-function-not-differentiable-at-the-rationals-differentiable-elsewhere/
ANALYSIS A CONTINUOUS FUNCTION NOT DIFFERENTIABLE AT THE RATIONALS BUT DIFFERENTIABLE ELSEWHERE NOVEMBER 30, 2014 JEAN-PIERRE MERX Math Counterexamples
(抜粋)
We build here a continuous function of one real variable whose derivative exists on R?Q and doesn’t have a left or right derivative on each point of Q.

As Q is (infinitely) countable, we can find a bijection n→rn from N to Q. We now reuse the function f defined here.
http://www.mathcounterexamples.net/a-differentiable-function-except-at-point-with-bounded-derivative
Recall f main properties:

This proves that hh is differentiable at aa with h′(a)=limn→+∞h′n(a). For a∈Q, we can find p∈N with a=rp.
Following a similar proof than above, the function lp:x→h(x)−up(x) is differentiable at a.
As f does not have left and right derivatives at 00, upup does not have left and right derivatives at a.
finally, the equality h=lp+up implies that hh also does not have left and right derivatives at a.

Conclusion:
the function h is differentiable at all irrational points but does not have left or right derivative at all rational points.
(引用終り)
0507132人目の素数さん
垢版 |
2017/12/24(日) 20:59:42.02ID:ndfap2+C
>>504
>そろそろ例の証明を読んでみたらどうだね。
同感です
0508現代数学の系譜 雑談 古典ガロア理論も読む ◆e.a0E5TtKE
垢版 |
2017/12/24(日) 21:03:53.86ID:Q5UHveEY
>>505
ID:ndfap2+Cさん、どうも。スレ主です。

レスありがとう(^^
あなたは
(>>401)
"どっちもどっち
ID:KNjgsEZnはただの基地外"
と書いてくれた人かな?

>>・いま、考えている通常のRにおいて、「内点を持たない閉集合」とは、孤立する1点から成る集合にほかならない
>違うよ
>境界点だけということ
>境界点が孤立してなくてはいけないわけではない

ああ、あなたは、レベル高そうだな(^^
ちょっと考えてみるよ
0509132人目の素数さん
垢版 |
2017/12/24(日) 21:12:24.67ID:KzYVQeN8
>ああ、あなたは、レベル高そうだな(^^
と、一年生の授業に着いていけない落ちこぼれが申しております
0510現代数学の系譜 雑談 古典ガロア理論も読む ◆e.a0E5TtKE
垢版 |
2017/12/24(日) 21:16:17.46ID:Q5UHveEY
>>507
ID:ndfap2+Cさん、あなたはレベル高そうだから、聞くが

>>497より)
「特に、THEOREM 5 変形トマエ函数(Ruler Function)のような、有理数で不連続、無理数で連続なる函数では、
”there is a dense uncountable subset of the reals at each point of which g fails to satisfy a Lipschitz condition.”
だと
だから、(A)”a dense uncountable subset”で、リプシッツ連続は満たさないは、実現できている
では、なぜ、(B)”内点を持たない閉集合の高々可算和”は、実現することができないのか?
[15] Gerald Arthur Heuer先生の(A)と、定理1.7 (422 に書いた定理)の(B)との差!」
の私の疑問点について、あなたの解釈は?

別に分り易く書いてくれとは言わないが

書いてくれたことに、一定の納得がいって、定理1.7(>>489)が、成り立ちそうということが見えれば、証明を読むことはやぶさかではない

だが、反例がありそうな証明を読むことは、特に必要がある場合は別として、私はしない(教科書に載っている、あるいは投稿論文の定理は別として)
0512132人目の素数さん
垢版 |
2017/12/24(日) 21:42:31.44ID:ndfap2+C
>>508
違うよ
0513現代数学の系譜 雑談 古典ガロア理論も読む ◆e.a0E5TtKE
垢版 |
2017/12/24(日) 21:53:33.96ID:Q5UHveEY
>>501-502
おっちゃん、どうも、スレ主です。
レスありがとう

あの・・、補集合 R−Bf というのは、平たくいうと、リプシッツ連続でなく、” |f(x)−f(y)|≦K|x−y| ”とできない場合だよね
で、これは、R上で稠密であってはならない

なぜならば、下記系1.8の証明で、有理数Qが、 1 点集合{p}の可算和であること、及び、稠密性から連続した区間(a, b) 内に必ず有理点{p}を含むという性質を使う
だから、もし、補集合 R−Bfが、R上で稠密でなら、同じ理屈で、区間(a, b) 内に必ず補集合 R−Bfの要素が存在することになり、定理の結論と矛盾するよ

(参考)
>>490
証明のPDFから、( https://www.axfc.net/u/3870548?key=Lipschitz 「定理1.7 (422 に書いた定理)」の証明PDF )
系1.8の証明で
「定理1.7 のBf について,
R − Q = (無理数全体) = (f の微分可能点全体) ⊆ Bf
が成り立つので,
R − Bf ⊆ Q = ∪ p∈Q {p}   (1)
である. ここで, 1 点集合{p} (p ∈ Q) は全部で可算無限個あり, 各{p} は内点を持たない閉集合であ
るから, (1) の右辺は内点を持たない閉集合の可算和である. よって, 定理1.7 が使えて, f はある開
区間(a, b) の上でリプシッツ連続である. 特に, f は(a, b) の上で連続である (2)
さて, Q はR 上で稠密だから, (a, b) ∩ Q ≠ Φ である. そこで, x ∈ (a, b) ∩ Q を何でもいいから1 つ取る.
(2) より,f は点x で連続であるが, 一方で, x ∈ Q とf の仮定により, f は点x で不連続である. これは矛盾. よって, 題意が成り立つ.」
0514現代数学の系譜 雑談 古典ガロア理論も読む ◆e.a0E5TtKE
垢版 |
2017/12/24(日) 22:01:49.91ID:Q5UHveEY
>>512
違うのか! それは残念だな(^^

ところで、>>513 に引用したけど、
” ここで, 1 点集合{p} (p ∈ Q) は全部で可算無限個あり, 各{p} は内点を持たない閉集合である”は、良いんだろ?

で、1 点集合以外で、R上において「内点を持たない閉集合」としては、どんな例があるのかな?( >>505より )
0516132人目の素数さん
垢版 |
2017/12/24(日) 22:11:48.09ID:ThBjkOXn
>>513
>あの・・、補集合 R−Bf というのは、平たくいうと、リプシッツ連続でなく、” |f(x)−f(y)|≦K|x−y| ”とできない場合だよね

補集合 R−Bf というのは、Af(x)=+∞ が成り立つ x の集合のこと。

” |f(x)−f(y)|≦K|x−y| ”

という曖昧な書き方では色々な誤解が入り込むので、そんな書き方をしてはいけない。
特にスレ主は、そんな書き方をしてはいけない。
繰り返すが、補集合 R−Bf というのは、Af(x)=+∞ が成り立つ x の集合のこと。


>で、これは、R上で稠密であってはならない
>(中略)
>補集合 R−Bfが、R上で稠密でなら、同じ理屈で、区間(a, b) 内に必ず補集合 R−Bfの要素が存在することになり、定理の結論と矛盾するよ

息をするように間違えるゴミクズ。

R−Bf が R 上で稠密であり、なおかつ「 R−Bf が第一類集合」が成り立つならば、
例の定理が適用できて、スレ主の指摘と合わせて矛盾が起きるので、その場合は
スレ主の言うとおりのストーリーになる。
しかし、R−Bf が R 上で稠密であっても、「 R−Bf が第一類集合」であるとは限らないので、
その場合は、例の定理がそもそも適用できず、スレ主のストーリーは破綻する。

つまり、今回のスレ主の勘違いは、「稠密なら自動的に第一類集合である」と勘違いしているところ。
0517132人目の素数さん
垢版 |
2017/12/24(日) 22:17:44.21ID:ThBjkOXn
これは俺からレスを書くと横レスになってしまうが、一応書いておく。

>>514
>で、1 点集合以外で、R上において「内点を持たない閉集合」としては、どんな例があるのかな?( >>505より )

ご老人よ、「カントール集合が該当する」と既に2,3回も書いているぞ(たとえば>>503)。

・ カントール集合は内点を持たない閉集合である。

・ もしカントール集合が一点集合の可算無限和で表現できるなら、
  第一類集合の観点からは一点集合を考えているのと同じことになってしまうが、
  実際にはカントール集合は非可算無限集合なので、そのようには表せない。

・ すなわち、カントール集合はスレ主の質問に対する明確な回答である。
0519132人目の素数さん
垢版 |
2017/12/24(日) 23:30:26.62ID:ndfap2+C
>>514
[0,1]-∪[0<i<n](i/n-1/n^3,i/n+1/n^3)
はどうかな
0520現代数学の系譜 雑談 古典ガロア理論も読む ◆e.a0E5TtKE
垢版 |
2017/12/24(日) 23:58:13.08ID:Q5UHveEY
>>517 & >>519

定理の定義を、カントール集合まで拡張しようというのかね?

https://www.axfc.net/u/3870548?key=Lipschitz 「定理1.7 (422 に書いた定理)」の証明PDF より)
「定義1.2 (X,O) は位相空間とする. S ⊆ X は, 高々可算無限個の閉集合Fi ⊆ X が存在して,
各Fiは内点を持たない,
S ⊆ ∪iFi
が成り立っているとする. このとき,「S は内点を持たない閉集合の高々可算和で被覆できる」と書くことにする.」

だったよね?

Fiとして、"一つのカントール集合"を許す?

そうすると、”個数”の数え方があいまいになるだろ?

”S ⊆ ∪iFi”で、Sは集合濃度で連続まで許すのか?
0521132人目の素数さん
垢版 |
2017/12/25(月) 00:05:24.76ID:BjcfoCpO
>>520
> >>517 & >>519
>
> 定理の定義を、カントール集合まで拡張しようというのかね?
>
> ( https://www.axfc.net/u/3870548?key=Lipschitz 「定理1.7 (422 に書いた定理)」の証明PDF より)
> 「定義1.2 (X,O) は位相空間とする. S ⊆ X は, 高々可算無限個の閉集合Fi ⊆ X が存在して,
> 各Fiは内点を持たない,
> S ⊆ ∪iFi
> が成り立っているとする. このとき,「S は内点を持たない閉集合の高々可算和で被覆できる」と書くことにする.」
>
> だったよね?
>
> Fiとして、"一つのカントール集合"を許す?
>
> そうすると、”個数”の数え方があいまいになるだろ?
>
> ”S ⊆ ∪iFi”で、Sは集合濃度で連続まで許すのか?


会話が成り立たないにもホドがあるだろw
0522132人目の素数さん
垢版 |
2017/12/25(月) 00:37:44.12ID:P3YrdrZj
>>520
>Fiとして、"一つのカントール集合"を許す?
当然ですよ
>そうすると、”個数”の数え方があいまいになるだろ?
どうして?
カントール集合で``1個''です
>”S ⊆ ∪iFi”で、Sは集合濃度で連続まで許すのか?
当然ですよ
0524132人目の素数さん
垢版 |
2017/12/25(月) 07:23:50.32ID:U1NU7yFp
>>520
既にレスしてくれている人が居るが、俺からもレスしておく。

>定理の定義を、カントール集合まで拡張しようというのかね?

拡張ではなく、最初からそういう適用が可能であるような定義になっている。
定義をキチンと読み直せ。もはや数学ではなく国語の問題である。
0525132人目の素数さん
垢版 |
2017/12/25(月) 07:29:02.91ID:U1NU7yFp
>>520
あるいは、権威主義のスレ主のために、次のような言い方をしてもよい。
まず、>>503 で書いたことを もう一度書くが、集合 A が内点を持たない閉集合の高々可算和で
被覆できるとき、A のことを「第一類集合」と呼ぶのである。従って、例の pdf の

> 「定義1.2 (X,O) は位相空間とする. S ⊆ X は, 高々可算無限個の閉集合Fi ⊆ X が存在して,
> 各Fiは内点を持たない,
> S ⊆ ∪iFi
> が成り立っているとする. このとき,「S は内点を持たない閉集合の高々可算和で被覆できる」と書くことにする.」

この記述は、「 S は第一類集合 」の定義を書いているだけである。
これとスレ主のトンチンカンな主張を組み合わせると、

「定義1.2 の集合 S は、各 F_i が高々可算無限集合でなければ第一類集合とは呼ばない( F_i に連続濃度を許すと、個数が曖昧になる)」

というアホな主張をしていることになる。しかし、第一類集合 S であって、
F_i を可算無限に限定することが出来ないものが ごく普通に存在するので、
この時点でスレ主は間違っていることになる。

ま、いずれにしても本質的には「国語の問題」なので、
スレ主はキチンと定義を読み直すことだ。
0526現代数学の系譜 雑談 古典ガロア理論も読む ◆e.a0E5TtKE
垢版 |
2017/12/25(月) 07:58:39.31ID:R/y0B5bE
>>521-522
>>カントール集合で``1個''です
>”S ⊆ ∪iFi”で、Sは集合濃度で連続まで許すのか?
>当然ですよ

なんだよ(^^
早く言ってくれればよかったのに(^^

でな、下記

リウヴィル数は、非可算集合、実数内で稠密で、ルベーグ測度は 0 であるから、内点を持たない
リウヴィル数の各点は、閉集合だと思うが、それで良いかな?

で、いま問題のRuler Functionでは、リウヴィル数が鬼門で
”not Lipschitzian at the Liouville numbers, for every r > 0”なんだよ

つまり、r→∞にしても、リウヴィル数以外の無理数は、Lipschitzianになるが、at the Liouville numbersではだめだと
で、そうすると、定理1.7 (422 に書いた定理)の反例になってないか?

(>>151)
https://ja.wikipedia.org/wiki/%E3%83%AA%E3%82%A6%E3%83%B4%E3%82%A3%E3%83%AB%E6%95%B0
リウヴィル数
(抜粋)
・リウヴィル数全体からなる集合は非可算集合であり、実数内で稠密であるが、1次元ルベーグ測度は 0 である。

http://mathforum.org/kb/message.jspa?messageID=5432910>>35より)
Topic: Differentiability of the Ruler Function Dave L. Renfro Posted: Dec 13, 2006 Replies: 3 Last Post: Jan 10, 2007
(>>494)
(抜粋)
THEOREM 2: The function f^r is: (B) continuous but not Lipschitzian at the Liouville numbers, for every r > 0;

(>>492
(抜粋)
Using ruler-like functions that "damp-out" quicker
than any power of f gives behavior that one would
expect from the above.

Let w:Z+ --> Z+ be an increasing function that
eventually majorizes every power function. Define
f_w(x) = 0 for x irrational, f_w(0) = 1, and
f_w(p/q) = 1/w(q) where p and q are relatively
prime integers.

** f_w is differentiable on a set whose complement
has Hausdorff dimension zero. Jurek [4] (pp. 24-25)

Interesting, each of the sets of points where these
functions fail to be differentiable is large in the
sense of Baire category.

つづく
0527現代数学の系譜 雑談 古典ガロア理論も読む ◆e.a0E5TtKE
垢版 |
2017/12/25(月) 07:59:20.31ID:R/y0B5bE
>>526 つづき

THEOREM: Let g be continuous and discontinuous on sets
of points that are each dense in the reals.
Then g fails to have a derivative on a
co-meager (residual) set of points. In fact,
g fails to satisfy a pointwise Lipschitz
condition, a pointwise Holder condition,
or even any specified pointwise modulus of
continuity condition on a co-meager set.

(Each co-meager set has c points in every interval.)
(引用終り)
以上
0528132人目の素数さん
垢版 |
2017/12/25(月) 08:20:37.64ID:nNJMc22f
スレ主 国語 国語
0531132人目の素数さん
垢版 |
2017/12/25(月) 17:03:43.01ID:P3YrdrZj
>>529
0532132人目の素数さん
垢版 |
2017/12/25(月) 17:11:47.85ID:P3YrdrZj
>>526
最初からそういう定義なんです
ところで
その関数の微分可能点が無理数の一部分なのですね?
そしてその補集合の濃度が非可算であると?
実際にどういう集合か分かりませんが
可算集合→可算個の疎な閉集合で覆える
は当たり前ですが同値ではありませんので
``反例''になっている``根拠''としては薄いと思います
0534現代数学の系譜 雑談 古典ガロア理論も読む ◆e.a0E5TtKE
垢版 |
2017/12/25(月) 18:05:46.46ID:I8rwcj5/
>>532
「ぷふ」さん、どうも(^^

>最初からそういう定義なんです

ああ、そうなんですか? 定理を書いた人の話は、最初それに否定的だったように聞いた気がしたが・・。気のせいかな(^^

>その関数の微分可能点が無理数の一部分なのですね?

Yes!
(>>526より)
http://mathforum.org/kb/message.jspa?messageID=5432910>>35より)
Topic: Differentiability of the Ruler Function Dave L. Renfro Posted: Dec 13, 2006 Replies: 3 Last Post: Jan 10, 2007
(抜粋)
The ruler function f is defined by f(x) = 0 if x is
irrational, f(0) = 1, and f(x) = 1/q if x = p/q
where p and q are relatively prime integers with q > 0.
(引用終わり)

一つは、この上記f(トマエ関数)をr乗した関数を考えているわけです。なのでYes!(それで、rはいくらでも大きく取れる)
もう一つは、 f(x) = 1/q^rではなく いかなるq^rよりも早く増大する(つまり、いかなる1/q^rよりも早く減少する)関数
w(q) を取って、f(x) = 1/w(q) としましょうということ。でも、無理数点で”g fails to satisfy a pointwise Lipschitz condition”が残ると

ここらは、上記のURLを読んでもらう方が話は早いでしょう
(なお、>>527の”co-meager (residual)”は、ベールの範疇定理の用語と解しています。
 ”c points”がいまいち分らんですが・・(^^ )

以上
0535132人目の素数さん
垢版 |
2017/12/25(月) 18:38:17.34ID:U1NU7yFp
>>526
>リウヴィル数は、非可算集合、実数内で稠密で、ルベーグ測度は 0 であるから、内点を持たない
>リウヴィル数の各点は、閉集合だと思うが、それで良いかな?

息をするように間違えるゴミクズ。

リウヴィル数の全体を L と置く。お前の持ち出した例では、L ⊂ R−B_f が言えているに過ぎないので、
このままでは例の定理に帰着できず、全く反例になってない。

では、R−B_f ⊂ L が成り立つと仮定した場合はどうか。ここでは一般的に、
R−B_f ⊂ L が成り立つような任意の写像 f:R→R について考えることにする。

L は内点を持たない集合で、L は非可算無限集合である。
よって、もし L 自体が閉集合なら、L は内点を持たない閉集合「1つ」となるので、

「内点を持たない閉集合 F_i の高々可算無限和」… (1)

として F_1=L, F_i=φ (i≧2) を採用すれば、R−B_f ⊂ L という包含は

R−B_f ⊂ F_1

を意味することになる。特に、R−B_f は(1)の被覆ができていることになり、例の定理が適用できる。
しかし、L は R 上で稠密なので、既に議論されたことと同じことをすれば矛盾し、例の定理は間違いとなる。
しかし、実際には、L 自体は全く閉集合ではないので、L そのままでは、R−B_f について(1)の被覆が
出来ていることにならず、スレ主の目論見は失敗に終わる。

[続く]
0536132人目の素数さん
垢版 |
2017/12/25(月) 18:43:50.14ID:U1NU7yFp
[続き]

ここで、L の各点 p に対して、一元集合 {p} は閉集合であることに注意する。
そこで、各 p∈L を適当に番号づけて F_i={p} と置き直して、

L=∪_i F_i

と表すことを考える。もしこのような芸当が可能ならば、R−B_f ⊂ ∪_i F_i となるので、
やはり例の定理が適用できることになり、そして矛盾するので、例の定理は間違いとなる。
しかし、実際には、L が非可算無限集合であるがゆえ、{p} も非可算無限個となるので、
F_i={p} と置く場合の F_i は可算無限個に収まらず、よって、このような F_i の置き方では
L=∪_i F_i という表現はできない。スレ主の目論見は やはり失敗に終わる。

このように、仮に R−B_f ⊂ L が言えているとしても全く反例にならないのである。
しかも、実際にスレ主が持ち出した例は L ⊂ R−B_f が言えているに過ぎず、余計に反例になってない。
結局、今回のスレ主の間違いは、次の3つである。

・ L ⊂ R−B_f が言えているに過ぎないシロモノを持ち出してきても、ぜんぜん反例になってない。

・ 仮に R−B_f ⊂ L が言えているとしても、R−B_f について(1)の被覆が出来ないので、やはり反例になってない。

・ どうもスレ主は、F_i の置き方をキチンと意識してないがゆえに、いつの間にか可算無限個の F_i で
  L=∪_i F_i と表現できているように勘違いしている節がある。
0537132人目の素数さん
垢版 |
2017/12/25(月) 19:34:09.08ID:U1NU7yFp
補足:

リウヴィル数の全体を L と置いたのだったが、この L は第「二類」集合であることが知られている。よって、

L ⊂ R−B_f

が成り立つような任意の f:R→R に対して、R−B_f は例の被覆が絶対に不可能であることが自動的に従い、
よって例の定理の適用範囲外となる。

一方で、スレ主の持ち出した f^r (r>0, f はトマエ関数) に対して

L ⊂ R−B_{f^r}

が成り立つのだから、結局、これらの f^r は、例の定理の反例に「ならない」ことが確定する。
0539132人目の素数さん
垢版 |
2017/12/25(月) 19:49:34.68ID:U1NU7yFp
>>538
>出典は?

バカなの? L は R 上で稠密なんだよ?もし L が閉集合なら、L の稠密性と合わせて

L=R

になってしまって矛盾するだろうが。だから、L は閉集合では無いんだよバカタレ。
0540132人目の素数さん
垢版 |
2017/12/25(月) 20:03:05.73ID:U1NU7yFp
さて、スレ主が >>527 などで たびたび引用している

>THEOREM: Let g be continuous and discontinuous on sets
>of points that are each dense in the reals.
>Then g fails to have a derivative on a
>co-meager (residual) set of points. In fact,
>g fails to satisfy a pointwise Lipschitz
>condition, a pointwise Holder condition,
>or even any specified pointwise modulus of
>continuity condition on a co-meager set.

についてもコメントしておく。この定理で扱われている g は、

「ある co-meager set の上で、g は全く pointwise Lipschitz condition を満たさない」

と主張されている。そこで、そのような co-meager set を1つ取って A とでも置いておく。
よって、g は A 上で全く pointwise Lipschitz condition を満たさないことになる。すなわち、

A ⊂ R−B_g

が成り立つことになる。A は co-meager set だったから、R−B_g は例の被覆が絶対に不可能であることが
自動的に従う。よって、このような g は自動的に、例の定理の適用範囲外となる。

特に、スレ主の大好きな f^r 及び f_w は、例の定理の反例に「ならない」ことが確定する。
これにて、スレ主が反例として疑っていた例は悉く壊滅したw

そして、上記の理由は「例の定理を経由しない理由」であるため、スレ主が >>497 で求めていた
「見極め」として十分であろう。これにて、いよいよスレ主は、例の「たった2ページの証明」を
読まなければならなくなった。
0541現代数学の系譜 雑談 古典ガロア理論も読む ◆e.a0E5TtKE
垢版 |
2017/12/25(月) 20:18:59.66ID:R/y0B5bE
>>539
>L は R 上で稠密なんだよ?もし L が閉集合なら、L の稠密性と合わせて
>L=R
>になってしまって矛盾するだろうが。だから、L は閉集合では無いんだよバカタレ。

それなら、Qも閉集合ではないだろ
ならば、「系1.8 有理数の点で不連続、 無理数の点で微分可能となるf : R → R は存在しない」(>>498
に適用して良いのか?
0542132人目の素数さん
垢版 |
2017/12/25(月) 20:23:08.79ID:P3YrdrZj
>>541
Qも閉集合じゃありません
しかし可算集合なので
可算個の疎な閉集合で覆えるのですよ
0543132人目の素数さん
垢版 |
2017/12/25(月) 20:29:05.20ID:nNJMc22f
>>530
>おれは、あほバカで
お前はあほバカではない
救い様の無いあほバカだ
何故なら自分がどんだけあほバカかの自覚が無いし、人の助言に聞く耳持たない頑固者だからだ
0544132人目の素数さん
垢版 |
2017/12/25(月) 20:29:29.60ID:U1NU7yFp
>>541
>それなら、Qも閉集合ではないだろ

お前はどこまでバカなんだ?今まで一体なにを読んでいたのだ?

もし Q 自体が閉集合なら、F_1=Q, F_i=φ (i≧2) と置けば終わる話。
しかし、実際には、Q 自体は閉集合ではない。そこはその通り。

ではどうするか?

F_i の作り方を工夫すればいいのである。具体的には、Q の元を適当に番号づけて、
各 q∈Q に対して F_i={q} と置けばいいのである。Q は可算無限集合なので、
このように設定した F_i の個数も可算無限個に収まり、しかも

Q=∪_i F_i, 各 F_i は内点を持たない閉集合

と表せるのだから、例の定理が適用できる形になっているだろうが。
L の場合にこの芸当が不可能なのは、

・ L 自体は閉集合ではないので、F_1=L, F_i=φ (i≧2) という置き方は不可能。
・ F_i の作り方を工夫して、F_i={q} (q∈L) と置くことにすると、今度は L が
 非可算無限集合であるがゆえに、F_i が可算無限個に収まらず、この置き方でも失敗する。

という理由があるからだよ。

結局お前は、F_i を「どのように上手く取ればいいのか」を全然 意識してないから、
そういうトンチンカンな間違いに陥るんだよ。
0545132人目の素数さん
垢版 |
2017/12/25(月) 20:30:12.13ID:nNJMc22f
>例の時枝に、終止符を打っていただいて(^^
見え透いた自演はやめろ
見てるこっちが恥ずかしくなる
0546現代数学の系譜 雑談 古典ガロア理論も読む ◆e.a0E5TtKE
垢版 |
2017/12/25(月) 21:00:53.16ID:R/y0B5bE
>>542 >>544

どちらのレスでも良いけど・・

話を、区間[0,1]に取って
Q' = {q | 0<q<1 q∈Q} なる集合Q'を考える

Q' は閉集合ではないですか?
もし、Q' が開集合なら、その補集合 [0,1]−Q' (これは区間[0,1]内の無理数の集合と0と1から成る)が閉集合になりますから

どうでしょうか
0547132人目の素数さん
垢版 |
2017/12/25(月) 21:11:29.72ID:U1NU7yFp
>>546
そのレスが何を意図しているのか全く意味不明。

R の通常の位相をθと書く。A⊂R に対して、θから定まるA上の相対位相を θ|_A と書く。
・・・という記法のもとで回答すると、

・ その Q' は、位相空間 (R,θ) において開集合にも閉集合にもなってない。

・ その Q' は、位相空間 ( [0,1], θ|_{[0,1]} ) において開集合にも閉集合にもなってない。

・ 例の定理は、位相空間を (R, θ) に固定して記述している定理なので、
  ( [0,1], θ|_{[0,1]} ) を持ち出したところで意味が無い。

・ そもそも、そのレスが何を意図しているのか全く意味不明。

・ スレ主はわざと無視しているのだろうが、そもそもの話として、>>540 で書いたことにより、
 スレ主の大好きな f^r 及び f_w は、例の定理の反例に「ならない」ことが既に確定している。
0549132人目の素数さん
垢版 |
2017/12/25(月) 21:34:18.06ID:U1NU7yFp
>>548
>Q全体では、開集合だと言われましたね?

意味不明。俺は一度もそんなトンチンカンな発言をした記憶は無い。
どのレスのことを指しているのか、具体的なレス番とともに指摘せよ。

ついでなので、先に回答しておく。

>部分集合である区間(0,1)の有理数が、なぜ開集合にも閉集合にもなっていないのですか?

・ その Q' は、位相空間 (R,θ) において開集合にも閉集合にもなってない。

・ その Q' は、位相空間 ( [0,1], θ|_{[0,1]} ) において開集合にも閉集合にもなってない。

・ その Q' は、位相空間 ( (0,1), θ|_{(0,1)} ) において開集合にも閉集合にもなってない。

なぜ「ならない」のかを、( (0,1), θ|_{(0,1)} ) の場合に説明する。

[続く]
0550132人目の素数さん
垢版 |
2017/12/25(月) 21:36:25.58ID:U1NU7yFp
[続き]

Q' が ( (0,1), θ|_{(0,1)} ) において開集合だとすると、(R,θ)の開集合 V が存在して、

Q'=(0,1)∩V

が成り立つことになる(相対位相の定義)。(0,1)∩V は (R,θ) における開集合なので、
Q'=(0,1)∩V の左辺である Q' も、(R,θ) における開集合ということになるが、これは明らかに矛盾する。

次に、Q' が ( (0,1), θ|_{(0,1)} ) において閉集合だとすると、(R,θ)の閉集合 K が存在して、

Q'=(0,1)∩K

が成り立つことになる(相対位相の定義)。ここで、x=1/√2 と置き、x_n → x を満たす
(0,1) 内の有理数列 x_n を何でもいいから1つ取る。このとき、x_n∈Q' であるから、
x_n∈(0,1)∩K すなわち x_n∈K となる。x_n→x だったから、K が(R,θ)の閉集合だったことから
x∈K となる。また、明らかに x∈(0,1) である。よって、x∈(0,1)∩K となるので、
x∈Q' となる。しかし、x は無理数なので矛盾する。

以上より、Q' は位相空間 ( (0,1), θ|_{(0,1)} ) において開集合にも閉集合にもなってない。
0551132人目の素数さん
垢版 |
2017/12/25(月) 21:43:23.58ID:nNJMc22f
>意味不明。俺は一度もそんなトンチンカンな発言をした記憶は無い。
>どのレスのことを指しているのか、具体的なレス番とともに指摘せよ。
スレ主はリアル呆け老人
0552現代数学の系譜 雑談 古典ガロア理論も読む ◆e.a0E5TtKE
垢版 |
2017/12/25(月) 23:45:06.17ID:R/y0B5bE
>>549-550
>以上より、Q' は位相空間 ( (0,1), θ|_{(0,1)} ) において開集合にも閉集合にもなってない。

了解。下記(yahoo)だね
R中に稠密に分散されている場合は、「開集合にも閉集合にもならない」ってことだね
あなたは力があるね〜(^^

連続濃度まで許すということだったが(>>522)、
結局は、稠密にR中に分散されている場合は、
「内点を持たない閉集合の高々可算和で被覆」は、”孤立する1点から成る集合”(>>490)の被覆に戻るわけだ!(^^

ところで・・・・
https://detail.chiebukuro.yahoo.co.jp/qa/question_detail/q13160534703
(抜粋)
delyunoaloveさん2016/6/1600:35:49
有理数空間Qは開かつ閉集合ですか?

ベストアンサーに選ばれた回答
kousaku2038さん 2016/6/1612:21:16
全体が実数Rなら、有理数Qは開でも閉でもない。
普通に考えて開集合でないことは、qを有理数とし、それを含む開区間(q-ε,q+ε)を考えると、この区間には無理数が存在するので、Qに含まれることはない。
閉集合でないことは、√2に収束する有理数列が取れることから、すぐにわかる。
(引用終り)

つづく
0553現代数学の系譜 雑談 古典ガロア理論も読む ◆e.a0E5TtKE
垢版 |
2017/12/25(月) 23:47:20.19ID:R/y0B5bE
>>552 つづき

(>>303より)
”定理1.7 (422 に書いた定理)
・・・「R−Bf が内点を持たない閉集合の高々可算和」・・・”

で、「R−Bf が内点を持たない閉集合の高々可算和」とは、なんだろうかと考えていた・・、連続濃度まで許すということにもからんで

1)「R−Bf が内点を持たない閉集合の高々可算和」で、Rで稠密で無ければ・・、「f はある区間(a, b) 上でリプシッツ連続」は自明
2)「R−Bf が内点を持たない閉集合の高々可算和」で、Rで稠密であれば・・、「f はある区間(a, b) 上でリプシッツ連続」は取れない(このケースは不存在)

だから、定理1.7 (422 に書いた定理)の証明では、1)の場合の証明は、全く不要で
2)の場合を厚く書いて、何か矛盾が起きることをしっかり証明すべきだったのでは?
(例えば、そういう函数が存在しないか、あるいは、「R−Bf が内点を持たない閉集合の高々可算和」での被覆ができないとか)

重ねて言えば、2)の場合について、「定理1.7に抵触するので、不成立」では、循環論法ではないだろうか?
(例えば、証明中で、無造作に区間(y,x)を取ったり、いろんな計算をしているが、R−Bf が”Rで稠密”という条件下では、許されない計算をしていないかどうか・・?)

つづく
0554現代数学の系譜 雑談 古典ガロア理論も読む ◆e.a0E5TtKE
垢版 |
2017/12/25(月) 23:48:58.79ID:R/y0B5bE
>>553 つづき

さて、従来の定理との比較で

1)不連続点が、dense(稠密)の場合、http://mathforum.org/kb/message.jspa?messageID=5432910>>526)にあるように、
 ”g fails to have a derivative on a co-meager (residual) set of points. In fact, g fails to satisfy a pointwise Lipschitz condition,”とある
2)無理数で可微分、dense(稠密)な有理点のみ微分不可の函数は構成あり(>>506
  http://www.mathcounterexamples.net/a-continuous-function-not-differentiable-at-the-rationals-differentiable-elsewhere/
3)で、定理1.7 は、上記をリプシッツ連続(あるいはディニ微分)に、拡張した定理と見ることが出来る。
  つまり、Bfが、リプシッツ連続(あるいはディニ微分可)で、
  補集合たるR−Bfが稠密の場合、そういう函数が存在しないか、あるいは、( 1)のように)「R−Bf が内点を持たない閉集合の高々可算和」での被覆ができないとなるのだろうか?

なお、以前から言っているが、なぜ3)についての研究が、いままで無かったのか?
そのナゾもまだ解けない
(不成立?)

まあ、年末なので、ゆっくりやりましょう
1)の証明と対比して読まないといけないと思うので
(そうしないと、証明にギャップがあっても気付かないだろうね、おれの頭じゃ(^^ )

以上
0555132人目の素数さん
垢版 |
2017/12/26(火) 00:17:11.07ID:5+kOkN0j
>>546
> Q' は閉集合ではないですか?
> もし、Q' が開集合なら、その補集合 [0,1]−Q' (これは区間[0,1]内の無理数の集合と0と1から成る)が閉集合になりますから
もしかしたら開でなければ閉と誤解してるかもしれませんが
閉でなくても開とは限りませんし開でなくても閉とは言えませんよ
(0,1]のような単純な例Qのような稠密な例いろいろです
0556132人目の素数さん
垢版 |
2017/12/26(火) 00:26:12.28ID:BhzQ/YUm
>>552
>結局は、稠密にR中に分散されている場合は、
>「内点を持たない閉集合の高々可算和で被覆」は、”孤立する1点から成る集合”(>>490)の被覆に戻るわけだ!(^^

ぜんぜん戻らない。

例えば、全ての有理数に適当に番号をつけて q_1, q_2, q_3, … と表しておく。また、カントール集合を C としておく。
F_i:= C + q_i (i≧1) と置く。ただし、C + q_i は、C を q_i だけ平行移動した集合を表すものとする。
このとき、各 F_i は非可算無限集合である。また、各 F_i は内点を持たない閉集合である。ここで、
A=∪_i F_i と置くと、この A は R の中に稠密に分布することが分かる。さらに、

「 A ⊂ ∪_i F_i , 各 F_i は内点を持たない閉集合 」

という状況が(明らかに)成り立っている。従って、

(1)「 A は R の中に稠密に分布し、なおかつ、A は内点を持たない閉集合の高々可算無限和で被覆できる」

という状況が成り立っている。すると、スレ主の主張によれば、この A は「孤立する1点から成る集合」の可算無限和で
被覆できることになるが、実際にはそれは不可能である。なぜなら、もしそれが可能だったとすると、
別の可算無限個の F ' _i が存在して、

・ 各 F ' _i は一元集合である
・ A ⊂ ∪_i F ' _i が成り立つ

という状況が成り立つことになるが、∪_i F ' _i は高々可算無限集合であり、一方で A は非可算無限集合であるから、
A ⊂ ∪_i F ' _i という包含は矛盾している。よって、この A の場合は、(1)が成り立っているにも関わらず、
A を「孤立する1点から成る集合」の可算無限和で被覆することは不可能である。
0557132人目の素数さん
垢版 |
2017/12/26(火) 00:31:34.36ID:5+kOkN0j
>>553
そもそも濃度とは関係ない定義なのですよ
ただし可算なら条件の成立は自明というだけのことです
あと
楽に証明できればそれに越したことはないので
自明の場合をことさらに分別する必要はありません
0558132人目の素数さん
垢版 |
2017/12/26(火) 02:02:59.93ID:yKt8KVjU
>もしかしたら開でなければ閉と誤解してるかもしれませんが
それがわかってないってもう壊滅的レベルだな
0559132人目の素数さん
垢版 |
2017/12/26(火) 10:34:54.09ID:033xN+6V
おいスレ主、きちんとレスをしろよ
他人の発言をでっちあげたのかオマエ?



549 132人目の素数さん sage 2017/12/25(月) 21:34:18.06 ID:U1NU7yFp
>>548
>Q全体では、開集合だと言われましたね?

意味不明。俺は一度もそんなトンチンカンな発言をした記憶は無い。
どのレスのことを指しているのか、具体的なレス番とともに指摘せよ。
0561現代数学の系譜 雑談 古典ガロア理論も読む ◆e.a0E5TtKE
垢版 |
2017/12/26(火) 11:51:57.60ID:oeOow6Ma
>>557
「ぷふ」さん、どうもスレ主です。
レスありがとう

ちょっと質問して良いですか?

>>303より)
”定理1.7 (422 に書いた定理)
f : R → R とする.
Bf :={x ∈ R | lim sup y→x |(f(y) − f(x))/(y − x)|< +∞ }
と置く: もしR−Bf が内点を持たない閉集合の高々可算和で被覆できるならば、 f はある開区間の
上でリプシッツ連続である.
(以下証明の文言から)
よって、 f は(a, b) 上でリプシッツ連続である.”

1.ここで場合分けをする
 1)補集合R−Bfが、R中で稠密で無い場合:この場合は、どこかにBfを満たす区間(a, b)が取れる(べき)。そして、条件Bfが成り立つならば、リプシッツ連続である
 2)補集合R−Bfが、R中で稠密である場合:この場合は、どこにもBfを満たす区間(a, b)は、取れない。
 3)上記場合分けにおいて1)2)とも、ほぼ自明。1)2)とも、証明の必要がない。だから、定理1.7は、証明の必要がない自明なことしか言っていない

つづく
0562現代数学の系譜 雑談 古典ガロア理論も読む ◆e.a0E5TtKE
垢版 |
2017/12/26(火) 11:58:51.44ID:oeOow6Ma
>>561 つづき

2.で、「系1.8 有理数の点で不連続、 無理数の点で微分可能となるf : R → R は存在しない」(>>498
 (その証明(>>513)より)
「定理1.7 のBf について,
 略
 (1) の右辺は内点を持たない閉集合の可算和である.
 略
 f は(a, b) の上で連続である (2)
 略
 (2) より,f は点x で連続であるが, 一方で, x ∈ Q とf の仮定により, f は点x で不連続である. これは矛盾. よって, 題意が成り立つ.」

 この証明中で、そもそも、有理数の点 x ∈ Qは、Rで稠密であるから、”f は(a, b) の上で連続である”の不成立は、当然(リプシッツ連続も含め)(∵稠密な有理点で不連続ゆえ)
 なので、定理1.7による必要もなく、もともとこれ(”連続である(a, b)が取れない”)は自明。

 そして、この背理法による論法もおかしい。
 例えば、>>554に示したように、”無理数で可微分、dense(稠密)な有理点のみ微分不可の函数は構成あり”(>>506)で、
 この背理法の論法が正しいならば、「微分可能なある区間(a, b)が取れないから(取れるとすると矛盾するから)、このような関数は存在しない」という結論が、導かれてしまう(本来有理点は稠密であるから、この背理法の論法自身がおかしい)

3.で、要は、定理1.7と系1.8とにおいて、”dense(稠密)”という意識が、あまりに希薄になってしまっているように思うのですが・・?

如何ですかね?

以上
0563132人目の素数さん
垢版 |
2017/12/26(火) 12:28:32.91ID:bh2BICch
>>562
もともと
0564132人目の素数さん
垢版 |
2017/12/26(火) 12:39:46.63ID:bh2BICch
もともと取れないからこそ背理法が効くわけです
可算集合の補集合で微分可能→ある開区間で連続→矛盾→可算集合の補集合で微分可能ではない
という流れですよ
ある開区間で連続以降の論証に持ち込むのに

可算集合の補集合で微分可能→ある開区間で連続

の論証が最も重要です
0565132人目の素数さん
垢版 |
2017/12/26(火) 12:55:35.93ID:bh2BICch
>>562
>  例えば、>>554に示したように、”無理数で可微分、dense(稠密)な有理点のみ微分不可の函数は構成あり”(>>506)で、
>  この背理法の論法が正しいならば、「微分可能なある区間(a, b)が取れないから(取れるとすると矛盾するから)、このような関数は存在しない」という結論が、導かれてしまう(本来有理点は稠密であるから、この背理法の論法自身がおかしい)
その関数は連続関数なのでは?それに微分可能な区間が取れないということからはそのような関数の存在も許されるということしか言えませんよ
0566132人目の素数さん
垢版 |
2017/12/26(火) 12:57:59.52ID:bh2BICch
許されるは変でした
許されないとは言えない
ですか
0567現代数学の系譜 雑談 古典ガロア理論も読む ◆e.a0E5TtKE
垢版 |
2017/12/26(火) 13:49:33.95ID:oeOow6Ma
>>564
「ぷふ」さん、どうもスレ主です。
早速のレスありがとう(^^

>可算集合の補集合で微分可能→ある開区間で連続

ここを詳しく書くと
A:稠密可算集合Q(有理数)で不連続で、その補集合(無理数)で微分可能→B:(ある条件を満たせば、必ず(例え補集合が不連続であってかつ稠密であっても))ある開区間で連続(命題Aは”ある条件を満たす”)→矛盾

というわけですね

だが、命題「B:(ある条件を満たせば、必ず(例え補集合が不連続であり(定理1.7 ではリプシッツ不連続だが)かつ稠密であっても))ある開区間で連続」で、

キモは、”例え補集合が不連続であり(定理1.7 ではリプシッツ不連続)かつ稠密であっても”ってところが、証明できちんと言えているかどうかですよね

そういう目で、証明を見て行かないと、すら〜と流してしまうと、ギャップがあっても見えない

「ぷふ」さんの目で見て、そこはどうなんですかね?
0568現代数学の系譜 雑談 古典ガロア理論も読む ◆e.a0E5TtKE
垢版 |
2017/12/26(火) 13:51:56.73ID:oeOow6Ma
>>565-566
>その関数は連続関数なのでは?それに微分可能な区間が取れないということからはそのような関数の存在も許されないとは言えないということしか言えませんよ

いや、もちろん連続関数です。

”無理数で可微分、dense(稠密)な有理点のみ微分不可の函数は構成可能”
 ↓
では、”無理数で可微分、dense(稠密)な有理点のみリプシッツ不連続(あるいはディニ微分不可)の函数は構成可能”か?

例の定理1.7は、これを”構成不能”と証明したということですか?

(なんで、だれもいままで気づかなかった? 本当に”構成不能”が成り立っている? 新定理? どう思いますか? )
0569132人目の素数さん
垢版 |
2017/12/26(火) 17:53:13.99ID:O+kvrrVD
>>568
(なんで、だれもいままで気づかなかった? 本当に”構成不能”が成り立っている? 新定理? どう思いますか? )

おーい!!
スレ主が迷走してるぞ。誰か黄色い救急車を呼んでやってくれ!!
0570132人目の素数さん
垢版 |
2017/12/26(火) 17:53:30.15ID:EtXFZwYa
英語で数学ど突き漫才
0571132人目の素数さん
垢版 |
2017/12/26(火) 19:13:34.92ID:84+rbTu3
>>567
>>可算集合の補集合で微分可能→ある開区間で連続
>
>ここを詳しく書くと
>A:稠密可算集合Q(有理数)で不連続で、その補集合(無理数)で微分可能→B:(ある条件を満たせば、必ず(例え補集合が不連続であってかつ稠密であっても))ある開区間で連続(命題Aは”ある条件を満たす”)→矛盾
Qで不連続は不要です
(ある条件)とは?
0572132人目の素数さん
垢版 |
2017/12/26(火) 19:20:36.30ID:84+rbTu3
>>568
>”無理数で可微分、dense(稠密)な有理点のみ微分不可の函数は構成可能”
> ↓
>では、”無理数で可微分、dense(稠密)な有理点のみリプシッツ不連続(あるいはディニ微分不可)の函数は構成可能”か?
>
>例の定理1.7は、これを”構成不能”と証明したということですか?
無理数で可微分有理数で不連続な関数は存在しないという結論を導けます
ところでリプシッツ不連続とは?
0573132人目の素数さん
垢版 |
2017/12/26(火) 19:43:29.01ID:yKt8KVjU
スレ主は知恵遅れ
但し悪知恵だけは人並み以上に発達している
0574現代数学の系譜 雑談 古典ガロア理論も読む ◆e.a0E5TtKE
垢版 |
2017/12/26(火) 19:47:04.40ID:IBTJ7HPw
>>571
黄金の救急車ですか?(^^
ご苦労さまです(^^

>Qで不連続は不要です

同意です
なお、”不連続”は、もともとは、>>562の「系1.8 有理数の点で不連続、 無理数の点で微分可能となるf : R → R は存在しない」(>>498)に由来しますよ

>(ある条件)とは?

系1.8の証明のキーになる定理で
>>561の定理1.7 (422 に書いた定理)より
”f : R → R とする.
Bf :={x ∈ R | lim sup y→x |(f(y) − f(x))/(y − x)|< +∞ }
と置く: もしR−Bf が内点を持たない閉集合の高々可算和で被覆できるならば”
が条件です。

なお、定理1.7の結論命題は、「f はある開区間の上でリプシッツ連続である.」(>>561)です。

(なお、この定理1.7 については、>>561に批判のコメントを書いたので、見て頂ければ幸いです)
0575現代数学の系譜 雑談 古典ガロア理論も読む ◆e.a0E5TtKE
垢版 |
2017/12/26(火) 19:49:19.10ID:IBTJ7HPw
>>572

>無理数で可微分有理数で不連続な関数は存在しないという結論を導けます

ええ、その通りです。なお>>526
http://mathforum.org/kb/message.jspa?messageID=5432910>>35より)
Topic: Differentiability of the Ruler Function Dave L. Renfro Posted: Dec 13, 2006 Replies: 3 Last Post: Jan 10, 2007
に、そのような記述があることは、過去なんども紹介しています

>ところでリプシッツ不連続とは?

上記>>574 の定理1.7での Bf :={x ∈ R | lim sup y→x |(f(y) − f(x))/(y − x)|< +∞ }

に対する補集合 R−Bfが満たすべき性質を、都合上、俗に”リプシッツ不連続”と呼称させて頂きました

Bf :={x ∈ R | lim sup y→x |(f(y) − f(x))/(y − x)|< +∞ }が、”リプシッツ連続”であること(これの補集合)に対する呼称です
0576132人目の素数さん
垢版 |
2017/12/26(火) 19:55:00.31ID:84+rbTu3
>>574
ならば
>ここを詳しく書くと
>A:稠密可算集合Q(有理数)で不連続で、その補集合(無理数)で微分可能→B:(ある条件を満たせば、必ず(例え補集合が不連続であってかつ稠密であっても))ある開区間で連続(命題Aは”ある条件を満たす”)→矛盾
ではなくて
A:可算集合の補集合で微分可能→B:ある開区間で連続
ですよ
その条件はAによって満たされています
0577132人目の素数さん
垢版 |
2017/12/26(火) 20:02:52.40ID:84+rbTu3
>>575
>上記>>574 の定理1.7での Bf :={x ∈ R | lim sup y→x |(f(y) − f(x))/(y − x)|< +∞ }
>
>に対する補集合 R−Bfが満たすべき性質を、都合上、俗に”リプシッツ不連続”と呼称させて頂きました
つまり
xにおいて``リプシッツ不連続''とは
limsup[y→x] |(f(y)-f(x))/(y-x)|=+∞
ということですか
ならば
無理数で可微分有理数でリプシッツ不連続な関数は存在しないという結論を導けますよ
0578現代数学の系譜 雑談 古典ガロア理論も読む ◆e.a0E5TtKE
垢版 |
2017/12/26(火) 20:14:55.29ID:IBTJ7HPw
>>576
>A:可算集合の補集合で微分可能→B:ある開区間で連続

えーと、可算集合を本来の目的である有理数Qに取ります。有理数Qの稠密性から、ある開区間(a,b)中に必ず、有理数が存在します。
いま、仮定として、有理数で不連続な関数を考えます。ですので、ある開区間(a,b)で連続は言えません

が、無理数のいたるところで、微分可能な関数は可能です
(しかし、無理数の全てで微分可能な関数は、できない。これらは、>>575のURLの通りです。)
0579現代数学の系譜 雑談 古典ガロア理論も読む ◆e.a0E5TtKE
垢版 |
2017/12/26(火) 20:15:47.76ID:IBTJ7HPw
>>577
>無理数で可微分有理数でリプシッツ不連続な関数は存在しないという結論を導けますよ

なるほど
それは興味深いですね

出典がありますか? あれば読んでみたい
おっと、このスレには書かないで下さい。

このスレでアスキー文字制限で書かれた数学の証明は、
読みにくくてしかたないのでね(^^
0582132人目の素数さん
垢版 |
2017/12/26(火) 20:25:26.12ID:BhzQ/YUm
これは俺の方から書くと横レスになってしまうが、一応レスしておく。

>>561
>1.ここで場合分けをする
>1)補集合R−Bfが、R中で稠密で無い場合:この場合は、どこかにBfを満たす区間(a, b)が取れる(べき)。そして、条件Bfが成り立つならば、リプシッツ連続である

よく読むと微妙に間違っている。「どこかにBfを満たす区間(a, b)が取れる」という条件だけでは

「 f は(a,b)上の 全 体 で リプシッツ連続である」

ということは導けないので、これでは例の定理の結論が導けていない。
ただし、「 Bfを満たす区間(a, b)が取れる」という条件からは、

「 f は(a,b)内の あ る 小 さ な 部 分 区 間 の 上 で リプシッツ連続である」

ということが、例の定理の「開区間版」を考えることにより成り立つので、結局は例の定理の結論が導けることには なる。
ただし、この論法では「例の定理の開区間版」を経由しなければならないので、実質的には例の定理を丸ごと最初から
証明し直すのと同じことになってしまう。すなわち、(1)の手順では、何も証明が始まってないことになる。つまり、

「(1)の場合は自明であり、何も証明する必要がない」

というスレ主の発言は大間違いである。

[続く]
0583132人目の素数さん
垢版 |
2017/12/26(火) 20:34:25.02ID:BhzQ/YUm
[続き]

>2)補集合R−Bfが、R中で稠密である場合:この場合は、どこにもBfを満たす区間(a, b)は、取れない。
>3)上記場合分けにおいて1)2)とも、ほぼ自明。1)2)とも、証明の必要がない。だから、定理1.7は、証明の必要がない自明なことしか言っていない

ここは全てが間違っていて、理屈が滅茶苦茶である。
スレ主の(2),(3)の理屈を成り立たせるためには、例の定理の結論が

「どこにもBfを満たす区間(a, b)は取れない」

という結論になっていなければならない。もしこうなっていたら、(2)の場合は、
スレ主の言うように自明であり、証明の必要が無い。しかし、実際には、例の定理の結論は

「 f は ある(a, b)の上でリプシッツ連続である」

という結論なのだから、「(2)の場合は証明の必要が無い」などというスレ主の理屈は全く成り立っておらず、
何かを致命的に勘違いしている。もしかしたら、スレ主は次のような勘違いをしているのかもしれない。

「 (2)の場合、例の定理と組み合わせると、"そのような f は存在しない" ことになるので、
 存在しない f を考えるのは無意味なことであり、ゆえに、この(2)は証明の必要がない。」

これの何が勘違いなのかは明白である。(2)の議論はそもそも、例の定理を「証明する」という前提での議論であるのに、
そこで「例の定理と組み合わせると」などと言って例の定理を適用してしまうなら、

「例の定理を証明するという前提の議論で、例の定理を適用する」

という循環論法に陥っていることになり、これでは何がしたいのか意味不明なのだ。
0584132人目の素数さん
垢版 |
2017/12/26(火) 20:37:45.80ID:BhzQ/YUm
結局、ここでのスレ主の勘違いを簡潔に述べると、次のようになる。

(1)での勘違い:
「どこかにBfを満たす区間(a, b)が取れる」という条件だけでは、例の定理の結論は導けず、
結局は例の定理を最初から丸ごと証明しなければならないような事態に陥るのに、
「(1)の場合は自明であり、何も証明する必要がない」 などと勘違いした。

(2)での勘違い:
スレ主は例の定理の結論が何なのかを全く把握せずに、勝手にスレ主自身の手で
場合分けした挙句に、その場合分けによって導かれる結論を

「もともとの例の定理の結論である」

と勝手に勘違いしてしまい、

「ゆえに、この場合は証明の必要がない」

などとトンチンカンな間違いに陥った。もしくは、無意識のうちに
例の定理そのものを適用してしまうという循環論法に陥ったがゆえに、

「この場合は証明の必要がない」

などとトンチンカンな間違いに陥った。
0585132人目の素数さん
垢版 |
2017/12/26(火) 20:37:56.49ID:84+rbTu3
>>578
有理数で不連続な関数→どの開区間でも連続ではない
無理数で微分可能な関数→ある開区間で連続
どちらも正しいということです
ところで
「無理数の至る所で微分可能な関数」はその前に書いている条件「有理数で不連続」も満たすのでしょうね?そしてその関数の微分可能な点の補集合は可算ではないということですね?
その関数に関しては「ある条件」が成り立たないでしょうね
成り立つとすれば矛盾を引き起こしますから
0586132人目の素数さん
垢版 |
2017/12/26(火) 20:39:54.53ID:84+rbTu3
>>579
ここで話題の定理の証明を読んでみてください
0587現代数学の系譜 雑談 古典ガロア理論も読む ◆e.a0E5TtKE
垢版 |
2017/12/26(火) 21:09:07.92ID:IBTJ7HPw
>>585
>>585
>有理数で不連続な関数→どの開区間でも連続ではない
>無理数で微分可能な関数→ある開区間で連続
>どちらも正しいということです

へー、どういうこと?

>「無理数の至る所で微分可能な関数」はその前に書いている条件「有理数で不連続」も満たすのでしょうね?そしてその関数の微分可能な点の補集合は可算ではないということですね?
>その関数に関しては「ある条件」が成り立たないでしょうね
>成り立つとすれば矛盾を引き起こしますから

同意です
上記のURLにあります
0589現代数学の系譜 雑談 古典ガロア理論も読む ◆e.a0E5TtKE
垢版 |
2017/12/26(火) 21:39:44.08ID:IBTJ7HPw
>>582
>よく読むと微妙に間違っている。「どこかにBfを満たす区間(a, b)が取れる」という条件だけでは
>「 f は(a,b)上の 全 体 で リプシッツ連続である」
>ということは導けないので、これでは例の定理の結論が導けていない

ああ、そうなのかい
それは、失礼した(^^

だが、系1.8の証明で「f はある開区間(a, b) の上でリプシッツ連続である. 特に, f は(a, b) の上で連続である」だった
だから、直に、「どこかにBfを満たす区間(a, b)が取れる」から、「特に, f は(a, b) の上で連続である」が言えるから、系1.8の証明にはそれで足りているだろ?
0590現代数学の系譜 雑談 古典ガロア理論も読む ◆e.a0E5TtKE
垢版 |
2017/12/26(火) 21:40:16.00ID:IBTJ7HPw
>>583

系1.8の証明で、
「Q はR 上で稠密だから, (a, b) ∩ Q ≠ Φ である. そこで, x ∈ (a, b) ∩ Q を何でもいいから1 つ取る.」という流れでしょ?

補集合R−Bfが、R中で稠密である場合は、同じ論法で、(a, b)の中に、補集合R−Bfの元が取れないのか?
0591132人目の素数さん
垢版 |
2017/12/26(火) 21:48:04.92ID:O+kvrrVD
▓ ▓ ▓ ▓ ▓ ▓ ▓ ▓
▓ ▓ ▓ ▓ ▓ ▓ ▓ ▓
▓ ▓ ▓ ▓ ▓ ▓ ▓ ▓
▓ ▓ ▓ ▓ ▓ ▓ ▓ ▓
▓ ▓ ▓ ▓ ▓ ▓ ▓ ▓ 👀
Rock54: Caution(BBR-MD5:0be15ced7fbdb9fdb4d0ce1929c1b82f)
0592132人目の素数さん
垢版 |
2017/12/26(火) 21:51:02.94ID:84+rbTu3
>>588
こう書くべきでしたか?

件の定理は無理数で可微分有理数でリプシッツ不連続な関数は存在しないという結論を導いていますよ
0593132人目の素数さん
垢版 |
2017/12/26(火) 21:54:27.76ID:84+rbTu3
>>587
>>有理数で不連続な関数→どの開区間でも連続ではない
>>無理数で微分可能な関数→ある開区間で連続
>>どちらも正しいということです
>
>へー、どういうこと?
どちらも正しいので無理数で微分可能有理数で不連続な関数が存在しないと結論できるわけです

>>「無理数の至る所で微分可能な関数」はその前に書いている条件「有理数で不連続」も満たすのでしょうね?そしてその関数の微分可能な点の補集合は可算ではないということですね?
>>その関数に関しては「ある条件」が成り立たないでしょうね
>>成り立つとすれば矛盾を引き起こしますから
>
>同意です
>上記のURLにあります
つまりその関数は件の定理の扱っている範疇外ということですね
0594132人目の素数さん
垢版 |
2017/12/26(火) 22:00:10.63ID:BhzQ/YUm
>>589
>だが、系1.8の証明で「f はある開区間(a, b) の上でリプシッツ連続である. 特に, f は(a, b) の上で連続である」だった
>だから、直に、「どこかにBfを満たす区間(a, b)が取れる」から、「特に, f は(a, b) の上で連続である」が言えるから、系1.8の証明にはそれで足りているだろ?

論理が滅茶苦茶。スレ主が>>561で主張していることは、あくまでも

「例の定理は証明の必要がない自明な定理だ」

というものである。俺はその主張に対して反論しているのである。
もし系1.8と絡めて「証明の必要がない自明な定理だ」という主張をしたいのであれば、
――――――――――――――――――――――――――
弱い定理:
f:R→R は、R−B_f が第一類集合であるとする。
このとき、f はある開区間の上で連続である。
――――――――――――――――――――――――――
という弱い定理を考えて、

「この "弱い定理" に関してなら、これは証明の必要がない自明な定理だ」

と主張するのが正しい手順である。

そして、スレ主の>>561の発言を "弱い定理" に差し替えて検証し直してみると、
スレ主の(1),(2),(3)のうち、(1)はスレ主の目論見通り、正しいことを言っていることになる。
しかし、(2),(3)が依然として滅茶苦茶であるから、結局、"弱い定理" に差し替えても
もスレ主の>>561の主張は間違っていることになる。
0595132人目の素数さん
垢版 |
2017/12/26(火) 22:10:18.09ID:BhzQ/YUm
>>590
>系1.8の証明で、
>「Q はR 上で稠密だから, (a, b) ∩ Q ≠ Φ である. そこで, x ∈ (a, b) ∩ Q を何でもいいから1 つ取る.」という流れでしょ?

>補集合R−Bfが、R中で稠密である場合は、同じ論法で、(a, b)の中に、補集合R−Bfの元が取れないのか?

取れるよ。R−Bf がR中で稠密である場合は、(a, b)の中に、R−Bf の元が取れるよ。
で?その論法を使うことによって、一体どうやって

「 f は ある(a, b)の上でリプシッツ連続である」

という結論を導くのだね?>>561におけるスレ主の最終的な目標は、

「 (2)の場合は自明なので証明の必要がない 」

という主張に持っていくことだろ?より丁寧に書けば、ここでのスレ主の最終的な目標は、

「 (2)の場合は、例の定理の結論が自明に従うので、このケースは証明の必要がない 」

という主張に持っていくことだろ? そのためには、(2)を使うことで

「 f は ある(a, b)の上でリプシッツ連続である」

という結論が自明に導けなくてはならないだろ?
それで、一体どうやって、(2)からこの結論を自明に導くのだね?

スレ主は(2)から一体何を「結論」しようとしているのだね?
スレ主は何かを盛大に勘違いしまくっているぞ?
0596132人目の素数さん
垢版 |
2017/12/26(火) 22:47:45.59ID:BhzQ/YUm
>>590
>系1.8の証明で、
>「Q はR 上で稠密だから, (a, b) ∩ Q ≠ Φ である. そこで, x ∈ (a, b) ∩ Q を何でもいいから1 つ取る.」という流れでしょ?

>補集合R−Bfが、R中で稠密である場合は、同じ論法で、(a, b)の中に、補集合R−Bfの元が取れないのか?

もしかして、スレ主はこういうことが言いたいのか?
―――――――――――――――――――――――――――――――――――――――――――――――――――――――
(2):稠密の場合は、どんな開区間(a,b)の中にも R−B_f の元が紛れ込んでしまうが、一方で例の定理によれば、
f はある開区間の上でリプシッツ連続なので矛盾する。よって、このケースはそもそも起こらないので考えなくてよい。
―――――――――――――――――――――――――――――――――――――――――――――――――――――――

もし、このような趣旨の発言をしているつもりならば、
それは>>583の後半で指摘したことと全く同じことであり、これでは何も言えてないぞ?

スレ主は、例の定理が自明であることを実証しようとしているのに、
その最中に例の定理そのものを適用してしまったら循環論法だぞ?
別の言い方をすると、上記の2行で言っていることは

「例の定理を適用すれば、例の定理は自明である」

というアホな発言なんだぞ?
スレ主は それで何を言ったつもりになってるんだ?
0597現代数学の系譜 雑談 古典ガロア理論も読む ◆e.a0E5TtKE
垢版 |
2017/12/26(火) 23:24:09.60ID:IBTJ7HPw
>>592
>こう書くべきでしたか?

いいえ

>件の定理は無理数で可微分有理数でリプシッツ不連続な関数は存在しないという結論を導いていますよ

それは証明を読まずとも分る
問題は、定理1.7 (422 に書いた定理)の数学的な意味を見極めて、それが数学的に意味があると分った場合にのみ証明を読むと。いま、途中です。そう焦らないで(^^
なお、繰返すが、>>561に批判のコメントを書いたので、見て頂ければ幸いです

ところで、貴方は博識みたいだから、聞くが
定理1.7 (422 に書いた定理)か、あるいは類似の定理でも良いが、どこか教科書か論文にありませんかね?
あれば、それを見てみたいのだが・・
0598現代数学の系譜 雑談 古典ガロア理論も読む ◆e.a0E5TtKE
垢版 |
2017/12/26(火) 23:25:21.75ID:IBTJ7HPw
>>593
>どちらも正しいので無理数で微分可能有理数で不連続な関数が存在しないと結論できるわけです

どこかに出典がありそうですね。
よければ、出典を教えて下さい
(ε近傍の話かな?)

>つまりその関数は件の定理の扱っている範疇外ということですね

そうでしょうね
0599現代数学の系譜 雑談 古典ガロア理論も読む ◆e.a0E5TtKE
垢版 |
2017/12/26(火) 23:26:09.73ID:IBTJ7HPw
>>596
>スレ主は それで何を言ったつもりになってるんだ?

単純に場合分けをしただけだよ(>>561を 微修正)
 1)補集合R−Bfが、R中で稠密で無い場合:この場合は、どこかにBfを満たす区間(a, b)が取れる(べき)。そして、条件Bfが成り立つならば、(a, b)で連続である
 2)補集合R−Bfが、R中で稠密である場合:この場合は、どこにもBfを満たす区間(a, b)は、取れない。
それだけ
■ このスレッドは過去ログ倉庫に格納されています